subject
Mathematics, 14.04.2021 17:40 hannah2718

Which statement is correct for the function graphed below? (A) There is a maximum value of 3.

(C) There is a minimum value of 3.

(B) There is a maximum value of 2.

(D) There is a minimum value of 2.

* The answer that you see on the picture is wrong *


Which statement is correct for the function graphed below?

(A) There is a maximum value of 3.
(C)

ansver
Answers: 2

Another question on Mathematics

question
Mathematics, 21.06.2019 16:40
The table shows the total distance that myra runs over different time periods. which describes myra’s distance as time increases? increasing decreasing zero constant
Answers: 2
question
Mathematics, 21.06.2019 18:00
Adj has a total of 1075 dance and rock songs on her system. the dance selection is 4 times the size of the rock selection. write a system of equations to represent the situation.
Answers: 1
question
Mathematics, 21.06.2019 19:20
1- what do you think the product of a nonzero rational number and an irrational number is? is it rational or irrational? make use of variables, the closure property of integers, and possibly a proof by contradiction to prove your hypothesis.2- why do we have to specify that the rational number must be nonzero when we determine what the product of a nonzero rational number and an irrational number is? if the rational number were 0, would it give us the same result we found in the first question?
Answers: 2
question
Mathematics, 21.06.2019 19:30
Mrs. gehrke said cheddar weighs 16.8 pounds. he actually weighs 15.2 pounds. what is the percent error?
Answers: 1
You know the right answer?
Which statement is correct for the function graphed below? (A) There is a maximum value of 3.
...
Questions
question
Mathematics, 24.01.2022 23:40
question
Mathematics, 24.01.2022 23:40
question
Mathematics, 24.01.2022 23:40
question
Mathematics, 24.01.2022 23:40
Questions on the website: 13722367